Mondejar - Digests - Art.440-465 PDF

You might also like

Download as pdf or txt
Download as pdf or txt
You are on page 1of 49

PROPERTY Digested by Jennylyn C.

Mondejar

Article 440 - 444

1. Tacas v. Tobon GR NO. 30240 Aug 23, 1929

Facts:

Francisco Dumadag owned three parcels of land. He later on died and immediately after his death, Evaristo
Tobon took possession of the three parcels of land planting them with tobacco. Since then, Tobon had been
collecting the fruits therefrom, which consisted of rice and tobacco.

Now, Aquilina Tacas and other heirs of Francisco filed an action to recover from the defendant
the ownership and possession of three parcels of land together with the fruits collected by him
during the time he was in possession of said land. They alleged that the defendant unlawfully took
said parcels upon the death of Francisco and that he remained in possession, enjoying the fruits to
the value of P700 annually.

Tobon contended that he is the owner of said land pursuant to a deed of sale from Exequiel Tacas,
the brother-in-law of Francisco.

RTC ruled in favor of the heirs of Francisco Dumadag and Tobon must give the sum of P11,040.00
which is the total value of rice and tobacco from 1912-1927.

Issue:

WON the restitution should be made since 1912 when Tobon collected the fruits of the land of the
late Tacas until the termination of the case

Ruling:

No. Even the possessor in good faith must return the fruits received from the time the answer to
the complaint was filed, that is, from the time he became aware that he was in undue possession.

In this case, Tobon should only be bound to return the fruits received from April 1918 (when he filed
his answer) to 1927, with the right to deduct the expenses of planting and harvesting pursuant to
then Article 365 of the Civil Code.
PROPERTY Digested by Jennylyn C. Mondejar

2. Bachrach Motor v. Talisay-Silay GR NO. 35223 Sep 17, 1931

Facts:

Talisay-Silay Milling Company is engaged in sugar manufacturing and had borrowed money from
Philippine National Bank. To secure payment, Talisay induced its planters to mortgage their
property in exchange of bonuses.

Now, one of its planters, Mariano Lacson Ledesma, was indebted to Bachrach Motor and then
failed to pay Bachrach. Bachrach averred that it is entitled to Lacson’s bonus. The Bank opposed
and claimed that the bonus is civil fruits of land mortgaged by Lacson in the bank.

Issue:

WON the bonus in question is civil fruits.

Ruling:

No. The bonus is not civil fruits because it is not immediate and merely accidental from the land
mortgaged by Lacson to Philippine National Bank in favor of Talisay.

The Court further explained that bonus bears no immediate, but only a remote accidental
relation to the land mentioned, having been granted as compensation for the risk of having
subjected one's land to a lien in favor of the bank, for the benefit of the entity granting said bonus.
Article 355 of the Civil Code considers three things as civil fruits: First, the rents of buildings;
second, the proceeds from leases of lands; and, third, the income from perpetual or life annuities,
or other similar sources of revenue.

If this bonus be income or civil fruits of anything, it is income arising from said risk, or, if one
chooses, from Mariano Lacson Ledesma’s generosity in facing the danger for the protection of the
central, but certainly it is not civil fruits or income from the mortgaged property, which, as far as this
case is concerned, has nothing to do with it.
PROPERTY Digested by Jennylyn C. Mondejar

3. Torbela v. Spouses Rosario GR NO. 140528 Dec 7, 2011

Facts:

This case involves a parcel of land owned by the Torbela siblings. They executed a deed of
absolute quitclaim covering said lot in favor of Dr. Rosario, who is the builder of the improvements
therein.

The Torbela siblings alleged that they allowed Dr. Rosario to register the lot in his name, however,
there is another deed of quitclaim this time in favor of the Torbela, acknowledging that he only
borrowed the lot so he could obtain a loan from DBP and that he will be returning the same after
said loan obligation be fulfilled.

He used the proceeds of the loan to construct a building on the lot. Upon fulfilling his obligation
with DBP, he contracted a second loan with PNB and subsequently with Banco Filipino. He failed
to pay his loan so the lot was extrajudicially foreclosed and was sold at a public auction.

Issue:

WON Dr. Rosario is a builder in good faith..

Ruling:

No. He is a builder in bad faith since he knew that the lot was not his. The right of accession is
recognized under Article 440 of the Civil Code which states that "[t]he ownership of property gives
the right by accession to everything which is produced thereby, or which is incorporated or
attached thereto, either naturally or artificially.”

In this case, the building constructed by Dr. Rosario is something that is attached or incorporated
to the lot of the Torbela siblings and by right of accession, the latter is presumed to be the owner of
such property unless Dr. Rosario can prove otherwise that he is in good faith.
PROPERTY Digested by Jennylyn C. Mondejar

4. NHA v. Manila Seedling G.R. No. 183543, June 20, 2016

Facts:

Petitioner is the owner of a 120-hectare piece of government property in Diliman, Quezon City,
reserved for the establishment of the National Government Center.

President Ferdinand Marcos, by virtue of Proclamation No. 16706, reserved a seven-hectare area
thereof and granted respondent usufructuary rights over it. Respondent occupied a total of 16
hectares, thereby exceeding the seven-hectare area it was allowed to occupy and leased the
excess to private tenants.

Now, President Ramos, pursuernt to EO 58, wants to get the lease part and would like to transfer
respondent on a more suitable place for them. Respondents invoked Proclamation No. 16706.

Issue:

WON petitioner is entitled to recover rent.

Ruling:

Yes. Petitoners can claim rent over the excess lot occupied by respondent since respondent had
no right to act beyond the confines of the seven-hectare area granted to it, and since it was fully
aware of this fact, its encroachment of nine additional hectares of petitioner's property rendered it a
possessor in bad faith as to the excess.

Article 443 provides: He who receives the fruits has the obligation to pay the expenses made by a
third person in their production, gathering, and preservation.

In this case, respondent shall be made to account for the fruits it received from the time it took
possession until the time it surrendered the excess to petitioner.
PROPERTY Digested by Jennylyn C. Mondejar

5. Provincial Assessor Of Agusan Del SuR Vs. Filipinas Palm Oil Plantation, Inc. G.R. No.
183416, October 05, 2016

FACTS:
Filipinas Palm Oil Plantation Inc. (Filipinas) is a private organization engaged in palm oil plantation
with a total land area of more than 7,000 hectares of National Development Company (NDC) lands
in Agusan del Sur.Subsequently, pursuant to the Agrarian Reform Law, NDC Lands were
transferred to Comprehensive Agrarian Reform Law beneficiaries who formed themselves as the
merged NGPI-NGEI Cooperatives. Filipinas entered into a lease contract agreement with NGPI-
NGEI.The Provincial-Assessor of Agusan del Sur assessed Filipinas' properties found within the
plantation area. The provincial assessor included to assess real property taxes against the
petitioner for roads, bridges, culverts, pipes and canals as these belonged to the cooperatives.

ISSUE:
WON improvements made by Filipinas belongs to the cooperatives, hence exempted from real
property taxes.

RULING:
Yes. Roads and other real properties that Filipinas constructed became permanent improvements
on the land owned by the NGPI-NGEI by right of accession. Despite the land being leased by
FIlipinas when the roads were constructed, the ownership of the improvement still belongs to
NGPI-NGEI.
Article 445. Whatever is built, planted or sown on the land of another and the improvements or
repairs made thereon, belong to the owner of the land.
As provided under Article 440 and 445 of the Civil Code, the land was owned by the cooperatives
at the time Filipinas built the roads. Hence, whatever was incorporated in the land, either naturally
or artificially, belonged to the NGPI-NGEI as the landowner.
In this case, NGPI-NGEI is the owner of the land being leased by the respondent. The LGC
exempts all real properties owned by cooperatives without distinction. The law does not provide
that tax exemption can only be availed if the property is used by the cooperative itself.
PROPERTY Digested by Jennylyn C. Mondejar

6. VICENTE STO. DOMINGO BERNARDO v. CATALINO BATACLAN GR No. 44606, Nov 28,
1938

FACTS:
A contract of sale was executed by Pastor Samonte and others over a parcel of land of about 90
hectares to Domingo Bernardo. Upon entering the premises, plaintiff found the defendant, Catalino
Bataclan, who appears to have been authorized by former owners, as far back as 1922, to clear
the land and make improvements thereon. As Bataclan was not a party in the civil case, plaintiff, on
June 11, 1931, instituted against him a civil case. In this case, plaintiff was declared owner but the
defendant was held to be a possessor in good faith, entitled for reimbursement in the total sum of
P1,642, for work done and improvements made.

The defendant states that he is a possessor in good faith and that the amount of P2,212 to which
he is entitled has not yet been paid to him. Therefore, he says, he has a right to retain the land in
accordance with the provisions of article 453 of the Civil Code.

ISSUE:

WON the defendant is entitled to the P2,212 worth of improvements spent.

RULING:

No. The Civil Code confirms certain time-honored principles of the law of property. One of these is
the principle of accession whereby the owner of property acquires not only that which it produces
but that which is united to it either naturally or artificially. (Art. 353.)

Whatever is built, planted or sown on the land of another, and the improvements or repairs made
thereon, belong to the owner of the land even if the planter, builder, of sower has acted in good
faith, when a conflict of rights arises between the owners and it becomes necessary to protect the
owner of the improvements without causing injustice to the owner of the land.
PROPERTY Digested by Jennylyn C. Mondejar

7. Rex Daclison vs. Eduardo Baytion, G.R. No. 219811 April 6, 2016

FACTS:
Baytion alleged that he was a co-owner of a parcel of land. As the administrator, he leased portions
of the property to third persons. A one-story building was erected on the property which was
divided into seven units or stalls. One of the stalls was leased to a certain Leonida Dela Cruz
(Leonida) and when lease expired, Daclison and other persons acting under her took possession
of the portion and occupied it without the prior knowledge and consent of Baytion. Since then,
Daclison had been occupying the contested portion and using it for his business of selling marble
and other finishing materials without paying anyting to Baytion.

Baytion, being fully aware of what happened, demanded Daclison to vacate it but went unheeded.
Daclison contended that the portion between the riprap constructed by the government and is
outside of the land co-owned by Baytion.

The MeTC dismissed the case on the ground that Baytion failed to include his siblings or his co-
owners, as plaintiffs in the case. Baytion appealed the case to the RTC, which ruled that the MeTC
lacked jurisdiction to decide the case because the allegations in the complaint failed to constitute a
case of forcible entry. The CA ruled in favor of Baytion.

ISSUE:

WON the contested portion of land belongs to Baytion


RULING:

No. The Supreme Court emphasized the use of the words “on” and “thereon” in Article 445,
which provides:

ARTICLE 445. Whatever is built, planted or sown on the land of another and the
improvements or repairs made thereon, belong to the owner of the land, subject to the
provisions of the following articles.
In the case at bar, this contested portion cannot be considered an accretion under Article 457 on
the New Civil Code. To begin with, the land came about not by reason of a gradual and
imperceptible deposit. The deposits were artificial and man-made and not the exclusive result of
the current from the creek adjacent to his property.
PROPERTY Digested by Jennylyn C. Mondejar

8. PACIFIC FARMS, INC vs. ESGUERRA G.R. No. L-21783 November 29, 1969

FACTS:

Insular Farms sold the subject buildings in favor of Pacific Farms. However, Insular has an unpaid
balance to Carried Lumber Company for the lumber and construction materials sold and delivered
to Insular Farms.

Pacific Farms, Inc. filed a third-party claim, asserting ownership over levied buildings which it
acquired from Insular Farms. Thereafter, sheriff proceeded with public auction. Meanwhile, Pacific
Farms filed a complaint against the Company and the sheriff with the court a quo, praying that
judgment be rendered, (a) declaring null and void the levy and judicial sale of the six buildings, and
(b) adjudging the defendants jointly and severally liable to the plaintiff in the sum of P2,000 by way
of actual damages and for such amount as the court may deem proper and just to impose by way
of exemplary damages and for costs of the suit. Trial Court annulled the levy but denied claim for
actual and exemplary damages.
ISSUE:
WON Pacific Farms as an owner of the subject buildings has to pay the unpaid balance of Insular
Farms

RULING:
Yes, The application by analogy of the rules of accession would suffice for a just adjudication.
Article 447 of the Civil Code provides:
The owner of the land who makes thereon personally or through another, plantings, constructions
or works with the materials of another, shall pay their value; and, if he acted in bad faith, he shall
also be obliged to the reparation of damages. The owner of the materials shall have the right to
remove them only in case he can do so without injury to the work constructed, or without the
plantings, constructions or works being destroyed. However, if the landowner acted in bad faith, the
owner of the materials may remove them in any event with a right to be indemnified for damages. It
was pointed out that the root of this controversy is Octaviano and that Wilson did not contribute to
the error. When he was constructing said improvements, he believed that he was constructing
them on his lot and not Jardinico’s.
However, the Court of Appeals erred in not applying the rights of Jardinico found in Article 448 of
the Civil Code and instead, made Pleasantville and CTTEI liable for the obligations of Wilson to
Jardinico.
PROPERTY Digested by Jennylyn C. Mondejar

9. Heirs of Orosa v. Migrino G.R. No. 99338-40

FACTS:
A case was filed to the RTC in favor of the heirs of Orosa to moved for execution of judgment. The
lower court directed the LRA to submit the property’s description for approval. However, the LRA
did not comply with the order because it alleged that the property had been decreed to Jose
Velasquez. Thereafter, Goldenrod filed a motion to intervene, alleging an interest in the case. The
lower court permitted Goldenrod to file its pleading.

ISSUE:
WON Goldenrod interest can be protected in a separate proceeding

RULING:
No, the legal interest which entitles a person to intervene in a suit must be actual and material,
direct and immediate. A party seeking to intervene in a pending case must show that he will either
gain or lose by the direct legal operation and effect of a judgment.
In the present case, Goldenrod has failed to meet this criteria and the lower court gravely abused
its discretion in permitting intervention after having overlooked this matter.
Under Article 447 of the Civil Code, the plaintiff in an action for quieting of title must at least have
equitable title to or an interest in the real property which is the subject matter of the action.
Evidence of Goldenrod’s capacity on this point is inexistent because Goldenrod is not asserting a
claim to the property.
PROPERTY Digested by Jennylyn C. Mondejar

10. Homeowners v. J.M. Tuason G.R. No. 203883


Facts:
A parcel of land was previously registered in the name of Tuason, which was sold by his authorized
representative, Gregorio Araneta, to the general public, in accordance with Subdivision Plan.
Tuason executed a Deed of Donation and Acceptance over its subdivisions open spaces, including
subject land, in favor of the City Government by virtue of the city ordinance. Block 494 was
developed by the Homeowners Association of Talayan and the City Government. Tuason failed to
pay the realty taxes, which resulted to the sale of the property in which the same was sold to him.
Tuason executed a Deed of Absolute Sale of Block 494 to Talayan Holdings and subsequently,
subdivided it into four lots, which made HATVI to file a complaint for annulment of sale,
cancellation of titles and mortgage, acceptance of donation and damages.

Issue:

Whether or not Tuason is an owner in bad faith of the said property which obliges him to be liable
for damages against HATVI.

Ruling:

No, the court held that Tuason was acting well within its rights when it sold the property to THI
which had the right to rely on what appears on the title covering the same. HATVI has neither
factual nor legal basis to question the sale thereof by the Quezon City government for tax
delinquency. After the expiration of the redemption period, after all, a property acquired pursuant to
a tax delinquency sale, like that purchased from a public auction sale, passes to the purchaser,
free from any encumbrance or third party claim not inscribed on the certificate of title. Also, having
purchased the property from J.M. Tuason, THI was likewise acting well within its rights to cause the
subdivision thereof, offer the same to the general public and to utilize the same as security for the
loan it obtained from Equitable Bank. Given that the property was purchased at a tax delinquency
sale, on the other hand, Equitable Bank cannot be likewise considered in bad faith when it primarily
relied on what appeared on the title over the property.
PROPERTY Digested by Jennylyn C. Mondejar

Article 448

11.Ignacio v. Hilario G.R. No. L-175

FACTS:
Elias Hilario and wife are the owners of a parcel of land, part rice-land and part residential.
The Ignacios conceding to the Hilarios the ownership of the houses and granaries built by them on
the residential portion with the rights of a possessor in good faith, in accordance with article 361 of
the Civil Code. The initial judgment renders the defendants to be entitled to hold the position of the
residential lot until after they are paid the actual market value of their houses and granaries erected
thereon, unless the Hilarios prefer to sell them said residential lot, in which case Ignacios shall pay
the Hilarios the proportionate value of the residential lot taking as basis the price paid for the whole
land.

ISSUE:
Whether or not Hilario can order the removal of the structures that Ignacio built

RULING:
No. Article 361 provides: The owner of land on which anything has been built, sown or
planted in good faith, shall have the right to appropriate as his own the work, sowing or planting,
after the payment of the indemnity stated in articles 453 and 454, or to oblige the one who built or
planted to pay the price of the land, and the one who sowed, the proper rent.
The Supreme Court discussed the right of retention. If the land owner chooses to appropriate what
was built, planted or sown, then he must pay the BPS. If the land owner does not pay the BPS right
away, the BPS is not obliged to give up possession of the property. The BPS has the right of
retention.
The second principle is: There is no other choice under Article 448 except options 1 and 2. Initially,
the land owner must choose between 1 and 2. He cannot put his own option provided under the
law.
PROPERTY Digested by Jennylyn C. Mondejar

12. Tayag v. Yuseco G.R. No. 14043

FACTS:
Even before 1930, Yuseco had been rendering services to Lim without compensation. Out of
gratitude, Lim offered 2 lots to Yuseco who then built a house.Atty. Yuseco contends that the 2 lots
were donated to him, but there was not evidence of such donation.
Subsequently, a lease contract for five years was then executed with Php120.00 rental fee per year
wherein Maria Lim was to pay all the taxes and that the non-payment of rent would be sufficient
cause to rescind the contract. Maria Lim sold the subject lots to her daughter, Tayag for
Php4,000.00. The petitioners asked Yuseco to remove their house or else pay Php120 per month.
An action of ejectment was later filed against Yuseco

ISSUE:
Whether or not Article 361 (Now Art. 448) of the Civil Code can be applied.

RULING:
Yes, Pursuant to Article 361 of the old Civil Code which states that: The owner of land on which
anything has been built, sown, or planted in good faith, shall be entitled to appropriate the things so
built, sown or planted, upon paying the compensation mentioned in Article 453 and 454, or to
compel the person who has built or planted to pay him the value of the land, and the person who
sowed thereon to pay the proper rent therefor.
The principle here is once a party, the land owner, has made his choice, he has duly informed the
court of his choice and the court already informed the other party that this is the choice, he can no
longer convert or change it. He has to stick to his choice. So, before he communicates to the court,
he can change his mind anytime but once it is communicated, that is his choice and if he does not
pay, the land improvements, according to the Section 546, the builder has the right of retention. He
can keep the property and stay there until he is paid.
There is this “limited right of removal” which is not in the provisions but was enunciated in a case.
PROPERTY Digested by Jennylyn C. Mondejar

13. Ignao v IAC G.R. NO. 72876

Facts:
Florencio Ignao and and his uncles were former co-owners of a land, which was later partitioned by
court order. After they decided to partition (1/3 for Isidro and 2/3 for Florencio), the part of the
house of Isidro was sitting on the portion that was allocated to Florencio.
Florencio Ignao says that Art. 448 does not apply – as rules of co-ownership should apply and not
Art.448 where a third person is involved.
Issue:
WON Article 448 applies in the case at hand
Held:
No. When the house was built, the parties were still co-owners. So one did not built on the land of
another but on the land co-owned with Isidro. It was only after partition that it was discovered that
the part of the land stood on the part of Florencio’s land.
In this case, Florencio to just sell the portion of the land where the encroachment was sitting on.
That was the best workable solution. The lower court cannot dictate upon the land owner. The right
of choice belongs to the land owner, not to the BPS nor the court.

In co-ownership and then partition, Article 448 is not directly applicable. But for purposes of
determining the rights of the parties after the partition with encroachment, then Article 448 can be
applied. But allow the land owner to choose, the court must not dictate.
PROPERTY Digested by Jennylyn C. Mondejar

14. Pecson v CA, Spouses Nuguid


Facts:
Pecson used to be the owner of a land until it was sold to Spouses Nuguid. Even before Spouses
Nuguid’s acquired said land, a commercial apartment was built by Pecson. Spouses Nuguid claims
that it is their apartment and it was not part of the sale and they have better claim as to its rent
revenue. Now Person contended that they have better right one the land as they are now
possessor of land, citing Art. 448
Issue:
WON Art. 448 can be applied.
Held:
No. Article 448 does not apply to a case where the owner of the land is the builder, sower, or
planter who then later loses ownership of the land by sale or donation. It does not apply to a case
where a person constructs a building on his own land, for then there can be no question as to good
or bad faith on the part of the builder.
In this case, Pecson was the owner as well as the builder who loses possession of the land
through public auction in failure to pay realty taxes. The SC said that on the provision on indemnity,
the Court can apply by analogy Article 448. Right of retention was present in this case. The amount
to be paid is not the cost of the construction but the current market value of the property built,
planted or sown.
PROPERTY Digested by Jennylyn C. Mondejar

15. Pleasantville Dev’t. Corp. v. CA


Facts:
Jardinico is owner of a certain land Lot 9. His neighbour, Kee, who is owner of Lot 8 erected
buildings on Jardinico’s lot, thinking the whole time that Lot 9 was his. Kee thought that way
because an agent of the subdivision erroneously pointed out with authority that lot 9 was lot 8.
Naturally Jardinico sued for imdemnity.
Issue:
WON Article 448 applies.
Held:
Yes. Article 448 refers to the options available to the land owner if the parties want to go to court.
Take note that the parties can enter into a compromise agreement.
The Supreme Court. further noted that this is a classic example of Article 448 because Kee did not
know that he was not building on his lot. Kee built on a wrong lot and thus, he was in good faith.
Again, remember that the parties can enter into an agreement. If you reach Civil Procedure, you
will learn that there is judgment upon a compromise. Even if you are already going to court, you
can enter into some kind of agreement.
PROPERTY Digested by Jennylyn C. Mondejar

16. Parilla v Pilar


Facts:
Parilla is a tenant over a parcel of land owned by Pilar. Parilla constructed buildings and claims
indemnity for it when the term of lease ended, as they were builders in good faith, citing Art 448.
Issue:
WON Art 448 can be applied.
Held:

No. Under Article 448, Parilla, who constructed the sari-sari store and the billiard hall, is
not entitled to reimbursement. The land owner here is not obliged to choose between the
two options in Article 448 because the said provision does not apply.
Further more the Court cited Art. 1678, If the lessee makes, in good faith, useful
improvements which are suitable to the use for which the lease is intended, without
altering the form or substance of the property leased, the lessor upon the termination of
the lease shall pay the lessee one-half of the value of the improvements at that time.
Should the lessor refuse to reimburse said amount, the lessee may remove the
improvements, even though the principal thing may suffer damage thereby. He shall not,
however, cause any more impairment upon the property leased than is necessary.
Parilla were not suitable to the use for which the lease was intended, then the lessor is not
obliged to pay 1⁄2 of the value of the improvements.
PROPERTY Digested by Jennylyn C. Mondejar

17. TECNOGAS PHILIPPINES MANUFACTURING CORP VS COURT OF APPEALS G.R. No.


108894. February 10, 1997

FACTS:
Technogas Phils is the registered owner of a parcel of land in Paranaque, purchased from Pariz
Industries together with all the buildings and improvements, including the wall existing thereon.
Respondent purchased a lot adjoining the petitioner’s land, upon learning of the encroachment by
its building and wall of a portion of the defendant’s land, the petitioner was offered to buy from the
defendant particular portion of 770 sq meters more or less but the defendant refused.
ISSUE:
WON petitioner is a builder in bad faith.
RULING:
NO. The time when to determine the good faith of the builder under Art 448 is reckoned during the
period when it was actually being built; and in a case where no evidence was presented nor
introduced as to the good faith or bad faith of the builder at that time, as in this case, he must be
presumed to be a builder in good faith, since bad faith cannot be presumed.
The current market value of the land and the value of the encroaching structure must be
determined because if option 2 is chosen, the value of the land and building must be identified.
When the land owner chooses option 1, the rentals will stop, there is now a right of retention.
Presumably, the land owner will now appropriate that portion and take possession of the part of the
building and the fence that encroach his property. Since it is now the owner of the land who wants
to appropriate the property, then it is he who has the obligation to pay in order to assume
possession of the property. Otherwise, Technogas now has the right of retention without having
been to pay rent because the option has been made.
If the second option is chosen, the rent must be paid until the builder (Technogas) pays the value
of the land.
PROPERTY Digested by Jennylyn C. Mondejar

18. Republic v. Judge Ballocanag GR NO.163794 Nov 28, 2008


FACTS: Reyes bought the subject land from Castillo and introduced extensive improvements,
planted fruit-bearing trees and employed many workers who regularly took care of the trees and
other plants. Reyes so prized this land which he bought and occupied in good faith for thirty years.
Unfortunately, a complaint for Cancellation of Title was filed by the Republic, as represented by the
Bureau of Forest Development. It was explained that the source OTC is spurious, fictitious and
irregularly issued.
Hence, the RTC rendered decision in favor of the Republic which was also affirmed by the CA. It
declared the OCT of Castillo and its derivative null and void, and declared the reversion of the land
in question to the government subject to the Agro-Forestry Farm Lease. Thus, aggrieved, Reyes
filed a Motion to Remove Improvements Introduced by Reyes on the Property.
ISSUE:
WON Reyes is entitled to the benefits of Article 448 of the Civil Code.
HELD:
Yes, Reyes is entitled to the benefits of Article 448 of the Civil Code. We have the landowner
(State) and the builder (Reyes).
However, there is a restricted application of Article 448. There is only one option in this case which
is number one. It is pointless to file a case for reversion.
In the instant case, the issue assumes full significance, because Articles 448 of the Civil Code
grant the builder or planter in good faith full reimbursement of useful improvements and retention of
the premises until reimbursement is made.
PROPERTY Digested by Jennylyn C. Mondejar

19. Sulo sa Nayon v. Nayong Pilipino GR NO.170923 Jan 20, 2009

FACTS:
Nayong Pilipino Foundation, a GOCC, is the owner of a parcel of land known as the Nayong
Pilipino Complex. Philippine Village Hotel, Inc. (PVHI), formerly called Sulo sa Nayon, Inc., is a
domestic corporation.
Nayong Pilipino leased a portion of the Nayong Pilipino Complex to Sulo sa Nayon, Inc. for the
construction and operation of a hotel building, to be known as the Philippine Village Hotel. The
hotel building and its other facilities was alleged to be worth about Php 2 billion pesos. The lease
was for an initial period of 21 years, or until May 1996. When the original lease agreement ended,
they agreed to the renewal of the contract for another 25 years.
However, Sulo sa Nayon defaulted in the payment of their monthly rental. Nayong Pilipino
Foundation repeatedly demanded to pay the arrears and vacate the premises but to no avail.
Hence, Nayong Pilipino filed a complaint for unlawful detainer before the MeTC of Pasay City.

What petitoners insists is that because of the improvements, which are of substantial value, that
they have introduced on the leased premises with the permission of Nayong Pilipino Foundation,
they should be considered builders in good faith who have the right to retain possession of the
property until reimbursement by Nayong Pilipino Foundation.
ISSUE:
WON Article 448 of the CC, is applicable.
HELD:
NO! The book of tolentino explains: Article 448 is manifestly intended to apply only to a case
where one builds, plants or sows on land in which he believes himself to have a claim of title,
and not to lands where the only interest of the builder, planter or sower is that of a holder such
as a tenant.
In the case at bar, Sulo sa Nayon have no adverse claim or title to the land. In fact, as lessees,
they recognize that Nayong Pilipino Foundation is the owner of the land. In fact, as lessees, they
recognize that the Nayong Pilipino Foundation is the owner of the land.

The Court affirms the ruling of the CA that introduction of valuable improvements on the leased
premises does not give the Sulo sa Nayon, Inc., et. al. the right of retention and reimbursement
which rightfully belongs to a builder in good faith. Otherwise, such a situation would allow the
lessee to easily "improve" the lessor out of its property.
PROPERTY Digested by Jennylyn C. Mondejar

20. Cabang v. Basay GR NO. 180587 Mar 20, 2009

Facts:
Cabang bought real property from the heirs of Felix Odong for P8, 000.00 but did not occupy the
said property.
Basay, on the other hand, had been in continuous, open, peaceful and adverse possession of the
same parcel of land since 1956 up to the present. They were the awardees in the cadastral
proceedings of Lot No. 7778. During the said cadastral proceedings, defendant-appellees claimed
Lot No. 7778 on the belief that they were actually occupying Lot No. 7777.
Petitoners filed a Complaint Recovery of Property against defendant-appellees.
Issue:
WON property subject of the controversy is a duly constituted family home which is subject to
execution.
Ruling:
Yes. It bears stressing that the purpose for which the records of the case were remanded to the
court of origin was for the enforcement of the appellate court’s final and executory judgment
As aptly pointed out by the appellate court, from the inception of Civil Case No. 99-20-127, it
was already of judicial notice that the improvements introduced by petitioners on the litigated
property are residential houses not family homes.

A family home to be considered as a family home should be built on the land owned by the builder.
Since the family home was built on the land of Basay, it cannot be considered a family home of
Cabang. Therefore, it can be a subject of execution. The land can be returned to Basay including
the house.

In this case, CA correctly ruled in favor of Basay when he filed a motion for execution to recover
the land including the house built by Cabang.
PROPERTY Digested by Jennylyn C. Mondejar

21. NARVAEZ vs. ALCISO G.R. No. 165907 July 27, 2009

Facts:
Larry A. Ogas owned a 1,329-square meter parcel of land and sold the property to his daughter
Rose O. Alciso (Alciso). Alciso entered into a Deed of Absolute Sale of property to Celso S. Bate
(Bate). Bate entered into a Deed of Sale of Realty, selling the property to the spouses Dominador
R. Narvaez and Lilia W. Narvaez (Spouses Narvaez). Meanwhile, Spouses Narvaez built a
commercial building on the property amounting to ₱300,000.

Later on, Alciso alleged that she informed the Spouses Narvaez that she wanted to repurchase the
property. The Spouses Narvaez demanded ₱300,000, but Alciso was willing to pay only ₱150,000.
Alciso and the Spouses Narvaez failed to reach an agreement on the repurchase price. In a
Complaint filed with the RTC, Alciso claimed that the intention of the parties was to enter into a
contract of real estate mortgage and not a contract of sale with right of repurchase.

Issue :
WON Art 448 is applicable.

Ruling :

NO, It applies only in cases where a person constructs a building on the land of another in good or
in bad faith, as the case may be. It does not apply to a case where a person constructs a building
on his own land, for then there can be no question as to good or bad faith on the part of the builder.

Article 448 is inapplicable in the present case because the Spouses Narvaez built the commercial
building on the land that they own. Besides, to compel them to buy the land, which they own, would
be absurd. What is applicable in this case is Article 1616 (deed of sale with a right to repurchase).

So, the price of the building plus the necessary and useful expenses. If Alciso wants to repurchase,
he has to pay first not under Article 448 but under Article 1616. Article 448 does not apply in deed
of sale with right to repurchase if the seller decides to repurchase the property.
PROPERTY Digested by Jennylyn C. Mondejar

22. TUATIS V. SPS. ESCOL GR No. 175399, October 27, 2009

Facts:
Escol (seller) and Tuatis (buyer) entered into a Deed of Sale by Installment of a part of a
Registered Land. Meanwhile, Tuatis already took possession of the subject property and
constructed a residential building thereon.

Later on, Ophelia requested Visminda to sign a Deed of Absolute Sale. However, Visminda refused
to do so contending that the purchase price was not yet fully paid

Subsequently,Tuatis failed to pay the remaining balance of six thousand pesos of the total
P10,000.00 purchase price of said property. As to Ophelia, she acted in bad faith for constructing a
residential building knowing that it was still Visminda who owned the land. On the other hand,
Visminda was in bad faith for allowing Ophelia to construct such building without opposition on her
part.

Issue :
WON Ophelia Tuatis, being the builder, is entitled to exercise the options granted in Article 448 of
the Civil Code.

Ruling :
No. Since it was already established that both of them acted in bad faith and their rights are to be
determined as though they have acted in good faith, Article 448 of the Civil Code is now applicable.

Under the first option, Visminda may appropriate for herself the building on the subject property
after indemnifying Ophelia for the necessary and useful expenses the latter incurred for said
building, as provided in Article 546 of the Civil Code. Until Visminda appropriately indemnifies
Ophelia for the building constructed, Ophelia has the right to retain possession of the building and
the subject property.

Under the second option, Visminda may choose not to appropriate the building and, instead, oblige
Ophelia to pay the present or current fair value of the land. However, if the present or current value
of the land, the subject property herein, turns out to be considerably more than that of the building
built thereon, Tuatis cannot be obliged to pay for the subject property, but she must pay Visminda
reasonable rent for the same. Visminda and Tuatis must agree on the terms of the lease;
otherwise, the court will fix the terms.

The rule that the choice under Article 448 of the Civil Code belongs to the owner of the land is in
accord with the principle of accession, i.e., that the accessory follows the principal and not the
other way around. Even as the option lies with the landowner, the grant to him, nevertheless, is
preclusive. The landowner cannot refuse to exercise either option and compel instead the owner of
the building to remove it from the land.
PROPERTY Digested by Jennylyn C. Mondejar

23. SPS. FUENTES V. ROCA GR No. 178902, April 21, 2010

Facts:

Tarciano father of the respondents offered to sell the lot to petitioners Manuel and Leticia Fuentes.
They arranged to meet at the office of Atty. Romulo Plagata whom they asked to prepare the
documents of sale. The agreement required the Fuentes Spouses to pay Tarciano a downpayment
of 60, 000 pesos and within 6 months Tarciano was to secure the consent of his estranged wife,
Rosario Roca, to the sale. According to Atty. Plagata, he went to see Rosario on one of his trips to
Manila and had her sign an affidavit of consent. In January 1989, Tarciano executed a deed of sale
in favor of the Fuentes Spouses. A new title was issued in the name of the Fuentes Spouses who
immediately constructed a building on the lot.

Eight years after the death of Tarciano and Rosario, their children, the Rocas, filed an action for
annulment of sale and reconveyance of the land against the Fuentes spouses claiming that the
sale was void because Rosario did not give her consent. They alleged that her signature on the
affidavit of consent had been forged. During the course of the trial, it was found that Rosario’s
signature on the affidavit of consent had been forged so the sale was declared void ab initio.
.
Issue :
Whether, Art. 448 is applicable

Ruling :
YES. A possessor in good faith is defined by Article 526 which states that, a possessor in good
faith is he who is not aware that there exist in his title or mode of acquisition any flaw which
invalidates it.

As possessor in good faith, the Fuentes spouses were under no obligation to pay for their stay on
the property prior to its legal interruption by a final judgment against them. What is more, they are
entitled under Article 448 to indemnity for the improvements they introduced into the property with
a right of retention until the reimbursement is made. The Rocas shall of course have the option,
pursuant to Article 546 of the Civil Code, of indemnifying the Fuentes spouses for the costs of the
improvements or paying the increase in value which the property may have acquired by reason of
such improvements. However, the builder or planter cannot be obliged to buy the land if its value is
considerably more than that of the building or trees. In such case, he shall pay reasonable rent, if
the owner of the land does not choose to appropriate the building or trees after proper indemnity.
The parties shall agree upon the terms of the lease and in case of disagreement, the court shall fix
the terms thereof. invalidates it.
PROPERTY Digested by Jennylyn C. Mondejar

24. FILOMENA R. BENEDICTO, Petitioner, vs. ANTONIO VILLAFLORES, Respondent. G.R.


No. 185020 October 6, 2010

Facts :

A property was originally owned by Maria Villaflores. She first sold it to Antonio Villaflores
(Villaflores). Antonio built his house on the property. A Deed of Sale was executed in favor of
Antonio, however, he did not have it registered. Fourteen years later, the same property was sold
to Filomena Benedicto (Benedicto). A Deed of Sale was executed in her favor. This time, Benedicto
registered it at the Registry of Deeds. Villaflores and Benedicto were now challenging the
ownership of the property. Pursuant to Article 1544 of the Civil Code, Benedicto was declared the
owner of the property since she was the first person who had it registered. However, Villaflores has
already built his house thereon. The Court of Appeals now seek to apply Article 448 to reimburse
him for the improvement.

Issue :
WON Article 448 is applicable.

Ruling :
YES, Antonio claims not being aware of any flaw in his title. He believed being the owner of the
subject premises on account of the Deed of Sale thereof in his favor despite his inability to register
the same. During the time he built his house, he fully believed that he was the owner of the
property even though he failed to have the sale registered.

The improvement was, in fact, introduced by Antonio prior to Filomena’s purchase of the land.
Thus, we sustain the finding that Antonio is a builder in good faith. Under Article 448, a landowner
is given the option to either appropriate the improvement as his own upon payment of the proper
amount of indemnity, or sell the land to the possessor in good faith. Relatedly, Article 546 provides
that a builder in good faith is entitled to full reimbursement for all the necessary and useful
expenses incurred; it also gives him right of retention until full reimbursement is made.
PROPERTY Digested by Jennylyn C. Mondejar

25. Communities Cagayan Inc. vs Spouses Nanol GR No. 176791 November 14, 2012

Facts:
Spouses Arsenio and Angeles Nanol entered into contract to Sell with petitioner Communities
Cagayan, Inc., whereby the former agreed to sell to respondent-spouses a house and Lots 17 and
19 for the price of P 368,000.00.
Respondent-spouses did not avail of petitioner’s in-house financing due to its high interest
rates. Instead, they obtained a loan from Capitol Development Bank, a sister company of
petitioner, using the property as collateral. To facilitate the loan, a simulated sale over the
property was executed by petitioner in favor of respondent-spouses. Unfortunately, the bank
collapsed and closed before it could release the loan.
Respondent-spouses entered into another Contract to Sell with petitioner over the same
property for the same price of P 368,000.00. This time, respondent-spouses availed of
petitioner’s in-house financing thus, undertaking to pay the loan over four years, from 1997
to 2001. Sometime in 2000, respondent Arsenio demolished the original house and
constructed a three-story house allegedly valued at P 3.5 million, more or less. In July 2001,
respondent Arsenio died, leaving his wife, herein respondent Angeles, to pay for the monthly
amortizations. However, there is a failure to pay amortizations.

Issue:
WON respondents are considered builders in good faith entitled to indemnification for necessary
and useful expenses and/or to buy the land under the provisions of the New Civil Code.

Held:
Yes. As a general rule, Article 448 on builders in good faith does not apply where there is a
contractual relation between the parties, such as in the instant case. In this case, the
parties failed to attach a copy of the Contract to Sell
In contract to sell, ownership is not transferred to the buyer because the land is still to be
amortized. Even if the property is delivered to the buyer, ownership still belongs to the
seller. When they improved the existing building here, they are not the owners and they
knew that they are not the owners because of the contract they entered into. They cannot
be considered builders in good faith but the seller (Communities) knew that the builder was
building but did not make oppositions and so they are also in bad faith.
Two bad faith equals good faith so Article 448 is applicable. There are many instances
where the other provisions of the Civil Code are involved. Sometimes, we have to
understand them as well in order to fully see the entire picture.
PROPERTY Digested by Jennylyn C. Mondejar

26. Leviste Vs. Legaspi GR NO. 199353 April 04, 2018

FACTS:
Legaspi Towers is a condominium building located at Paseo de Roxas, Makati City. Leviste
Management own the top floor of the condominium unit believing to be the owner of the "air" above
their floor and after acquiring the permission of the Condominium president, they started the
construction of a new floor. However, other unit owners fearing that the construction would affect
the structural integrity of the building refused the entry of the construction material.
Leviste Management filed an action that they may continue the construction. However, they
changed their mind and instead demanded that being builders in good faith they be indemnified
under Article 448 of the New Civil Code.

ISSUE:
WON Leviste Management may be indemnified under Article 448

RULING:
NO. Article 448 do NOT APPLY. This shall be governed by a special Law: Condominium Act. Under
the Condominium Act: the "Air" above the condominium is a common property among the
shareholders of the condominium, and that the parties are bound by a Master Deed where they are
required to conduct a meeting among the shareholders of the condominium prior the construction.
Leviste Management violated the master deed and is ordered to demolish the floor at their
expense.
Articles 448 and 546 of the Civil Code on builders in good faith are therefore inapplicable in cases
covered by the Condominium Act where the owner of the land and the builder are already bound
by specific legislation on the subject property (the Condominium Act), and by contract (the Master
Deed and the By-Laws of the condominium corporation).
PROPERTY Digested by Jennylyn C. Mondejar

27. Belvis v. Erola


G.R. No. 239727, July 24, 2019
Facts:

Conrado V. Erola, alleged owner of a lot covered by Transfer Certificate of Title and a tax
declaration was represented by their attorney-in-fact Maureen, filed a complaint for unlawful
detainer and damages. Respondents, further alleged that petitioner Cecilia Erola-Bevis were
allowed to possess the lot, subject to the condition that they would vacate upon demand, but when
asked to do so, petitioners refused.

On the contrary, petitioners claimed that the subject property was purchased by the late Rosario V.
Erola, the mother of petitioner Cecilia and respondent Conrado, however, Conrado allegedly
registered the property solely in his name. Hence, an implied trust was allegedly created over the
undivided hereditary share of petitioner Cecilia. Petitioners further alleged that for over 34 years,
they possessed and cultivated the lot in the concept of an owner, believing in good faith that they
were co-owners of the subject lot and introduced various improvements by planting bamboos, nipa
palms and coconut trees, and by constructing fishponds.

Issue:

WON Article 448 applies.

Ruling:

Yes. Petitioners have the right to retain the subject lot under Article 448 as the improvements were
built with the knowledge and consent of respondents.

However, there are cases where Article 448 of the Civil Code was applied beyond the recognized
and limited definition of good faith, e.g., cases wherein the builder has constructed improvements
on the land of another with the consent of the owner. The Court ruled therein that the structures
were built in good faith in those cases that the owners knew and approved of the construction of
improvements on the property. Despite being a possessor by mere tolerance, the DepEd is
considered a builder in good faith, since Cepeda permitted the construction of building and
improvements to conduct classes on his property.

In the case at bar, respondents as landowners have the following options: 1) they may appropriate
the improvements, after payment of indemnity representing the value of the improvements
introduced and the necessary, useful and luxurious expenses defrayed on the subject lots; or 2)
they may oblige petitioners to pay the price of the land, if the value is not considerably more than
that of the improvements and buildings. Should respondents opt to appropriate the improvements
made, however, petitioners may retain the subject lot until reimbursement for the necessary and
useful expenses have been made.
PROPERTY Digested by Jennylyn C. Mondejar

28. Parel v. Heirs of Prudencio G.R. No. 219640 January 15, 2020
Facts:

Deceased Simeon Prudencio is an owner of a two-storey residential house supported with a tax
declaration. That portion of a lot was registered in the name of Danilo Parel. The lot was previously
awarded to Danilo’s Father, who allowed Simeon to construct his house on a portion thereof. When
the house was finished, the Parel family was allowed to live there since they have no house of their
own. Danilo was able to acquire the lot through miscellaneous sales patent.

Issue:

WON the respondents are builders in good faith under Article 448.

Ruling:

YES. Under Article 448 of the Civil Code, the landowner is given two options: 1) he may
appropriate the improvements for himself after reimbursing the buyer the necessary and useful
expenses under Articles 546 and 548 of the Civil Code; or 2) he may sell the land to the buyer,
unless its value is considerably more than that of the improvements, in which case, the buyer shall
pay reasonable rent. As the landowner, Danilo cannot refuse to exercise his right of choice. He
cannot compel the builder to remove or demolish the improvement.

As ruled by the RTC and affirmed by the CA, Simeon was a builder in good faith: hence, the
provison of Article 448 of the Civil Code applies. In some special cases, the court has used Article
448 by recognizing good faith beyond the limited definition. Thus, the provision was applied to
cases wherein a builder had constructed improvements with the consent of the owner, as in this
case.
PROPERTY Digested by Jennylyn C. Mondejar

Articles 449-451

29. Congregation RVM v. CA GR No. 126363 Jun 26, 1998


Facts:

Congregation of the Religious of the Virgin Mary (RVM) purchased two lots — Lots 5-A and 5-C. In
between these lots was Lot 5-B which was not bought by the Congregation of the RVM in 1964.

In October of 1989, Spouses Protasio purchased Lot 5-B. In November 1989, after they had made
surveyed the lot, they discovered that Lot 5- B was already fenced and occupied by the RVM. They
also found out that a building for boy’s quarters and a portion of the RVM’s gymnasium building
were constructed inside Lot 5-B. Because of this, the spouses wanted to restore the possession of
Lot 5-B. Also, they demanded from the RVM to demolish the improvements constructed thereon
and to pay for damages.

The Congregation of the RVM admitted that they occupied Lot 5-B. Nonetheless, they claimed that
RVM built those improvements in good faith.

Issue:

Whether or not the Congregation of the RVM was a builder in good faith

Ruling:

No. The Congregation of the RVM was a builder in bad faith. Lot 5-B was already registered under
the name of the Protasio Spouses. The Congregation never gained title over the land and even
admitted that they never purchased such lot and, moreover, they admitted too that they introduced
improvements on the subject lot without the consent and knowledge of the spouses. Knowing that
the land did not belong to them, it follows that RVM was a builder in bad faith, so the Court ruled
that the spouses can invoke the remedy under Article 450 of the Civil Code. Pursuant to Article 451
in relation to Article 450 of the Civil Code, the landowner is entitled to damages from the builder.

It cannot be denied that appellant never gained title to the subject land as it admits to not having
purchased the said lot. Neither has appellant successfully shown any right to introduce
improvements on the said land. This being so, it follows that appellant was a builder in bad faith in
that, knowing that the land did not belong to it and that it had no right to build thereon, it
nevertheless caused the improvements in question to be erected. Therefore, Spouses Protasio
were entitled to recover possession of their property and to demand the demolition of the buildings
constructed thereon. However, they were not entitled to damages; the latter not being actually
proven and pleaded in their complaint.
PROPERTY Digested by Jennylyn C. Mondejar

30. DE VERA vs. CA and RICARDO RAMOS G.R. No. 97761. April 14, 1999.

FACTS:

Ricardo Ramos filed, a Complaint for recovery of property with damages against Agueda de Vera.
Ramos claims that he is the legal and absolute owner of a certain parcel of land known as Lot 2,
H-4-617.

Ramos said that de Vera is occupying a triangular portion of the property wherein they have
constructed a house of strong and permanent material.

Ramos demanded that the de Vera should remove their improvement thereon and vacate the said
portion, but the de Vera refused and failed, without any just or lawful cause to do so, to the present
time.

De Vera, et. al. theorized, that they have been in possession not only of 22 square meters but 70
square meters of land through their predecessor-in-interest, Teodoro de la Cruz husband of
Agueda De Vera and subsequently by themselves, as owners, before 1956.

ISSUE:

Whether or not de Vera, et. al. are possessors and builders in bad faith.

RULING: YES.

The Supreme Court held in this case that the facts and circumstances of this case are "outward
acts and proven conduct" indicating bad faith of petitioners as possessor and builder pursuant to
Articles 499, 450 and 451v0f the Civil Code.

In any event, the landowner is entitled to be indemnified by the builder in bad faith, pursuant to
Article 451 supra.

In the case at bar, Ricardo Ramos availed of the second alternative, which option is legally feasible
under the attendant facts and circumstances.
PROPERTY Digested by Jennylyn C. Mondejar

31. VERONA PADA-KILARIO and RICARDO KILARIO vs. COURT OF APPEALS and
SILVERIO PADA G.R. No. 134329. January 19, 2000.

FACTS:

Jacinto Pada permitted Feliciano Pada to construct a house on his land. When Feliciano died, his
son, Pastor, continued living in the house. Verona Pada-Kilario, one of Pastor’s children, and her
husband, Ricardo Kilario, have also been living in the house since 1960. Jacinto Pada died
intestate. His estate included a parcel of land of residential and coconut land.

Sometime in May, 1951, the heirs of Jacinto Pada entered into an extra-judicial partition of his
estate.

ISSUE:

Whether or not Pada-Kilaro are builders in good faith.

RULING:

NO.The Supreme Court ruled in this case that PADA-KILARO were in possession of the subject
property by sheer tolerance of its owners, they knew that their occupation of the premises may be
terminated any time considering that Spouses Kilario were in possession of the subject property by
sheer tolerance of its owners, they knew that their occupation of the premises may be terminated
any time. Thus, persons whose occupation of a realty is by sheer tolerance of its owners are not
possessors nor builders in good faith. Persons who occupy the land of another at the latter's
tolerance or permission, without any contract between them, is necessarily bound by an implied
promise that they will vacate the same upon demand, failing in which a summary action for
ejectment is the proper remedy against them.

In this case, the builders, Spouses Kilario, acted in bad faith and the landowner, Silverio, is in good
faith. Thus, Silverio has alternative rights granted under Articles 449-451 of the Civil Code. These
rights are:
1. He can appropriate to himself what has been built, planted, or sown without the obligation
to pay indemnity (Art. 449) plus he can demand damages (Art. 451)

2. He can demand the demolition of what has been built, planted, or sown all at the expense
of the BPS (Art. 450) plus damages suffered (Art. 451)

3. He can compel the builder or planter to pay the price of the land, and the sower the proper
rent (Art. 450), plus damages (Art. 451)
PROPERTY Digested by Jennylyn C. Mondejar

32. Heirs Durano vs Uy GR No. 136456 October 24, 2000


Facts:
A 128-hectare parcel of land were owned by Cebu Portland Cement Company (CEPOC). Said
property had been purchased by Durano & Co., Inc. On December 27, 1973, the late
Congressman Ramon Durano, Sr., together with his son Ramon Durano III, and the latter’s wife,
Elizabeth Hotchkiss Durano accused respondents officiating a "hate campaign”.
As early as the first week of August 1970, and even before many of the respondents received
notices to vacate, men who identified themselves as employees of Durano & Co. proceeded to
bulldoze the lands occupied by various respondents, destroying in their wake the plantings and
improvements made by the respondents therein. On some occasions, respondents alleged, these
men fired shots in the air, purportedly acting upon the instructions of petitioner Ramon Durano III
and/or Ramon Durano, Jr. On at least one instance, petitioners Ramon Durano III and Elizabeth
Hotchkiss Durano were seen on the site of the bulldozing operations.

Issue:
WON Durano were builders in bad faith.

Ruling:
YES. A purchaser of a parcel of land cannot close his eyes to facts which should put a reasonable
man upon his guard, such as when the property subject of the purchase is in the possession of
persons other than the seller. A buyer who could not have failed to know or discover that the land
sold to him was in the adverse possession of another is a buyer in bad faith.

In the case at bar, Uy, et. al are in possession and occupancy of the properties when Durano & Co. Inc.
supposedly purchased the same from CEPOC. The heirs of Durano made no attempt to investigate the nature of
Uy, et. al’s possession before they ordered the demolition in August 1970.

Also, the purchase of the property by Ramon Durano III from Durano & Co. Inc. could not be said to have been
made in good faith because:

1. Durano III acquired the property with full knowledge of Uy, et. al’s occupancy thereon;

2. There was undue haste in the conveyance of the property to Durano III, as the bulldozing operations were still
underway when the Deed of Sale was executed;

3. There was not even an indication that Durano & Co. Inc. attempted to transfer registration of the property in
its name before it conveyed the same to Durano III.

Since the heirs of Durano knew fully well the defect in their titles, they were correctly held by the Court of
Appeals to be builders in bad faith.
PROPERTY Digested by Jennylyn C. Mondejar

33. Padilla vs Malicsi GR No. 201354 September 21, 2016


Facts:
The spouses Padilla bought a parcel of land and later on discovered that Leopoldo Malicsi, Lito
Casino and Agrifino Guanes occupied said property. The Spouses had demanded through a verbal
and written demand to Malicsi and the others to vacate premises and pay a monthly rental of
P2,000 as monthly rental.
Spouses Padilla filed a complaint for recovery of possession. Malicsi, alleged that they believed in
all honesty and good faith that the lot belonged to Toribia Vda. De Mossessgeld. They claimed that
they possessed the land and built their houses on the lot only after receiving De Mossessgeld's
permission.
Issue:
Whether or not respondents have no right to recover their expenses over the improvements they
have introduced to petitioners' lot under Article 449 of the Civil Code.
Ruling:
No. Malicsi is not in good faith. The fact is that they never bothered to check who the owner is.
There was already a title in the name of Padilla which is a notice to the whole world that he owns
the property. Malicsi is in bad faith thus Padilla can avail of the option in Article 449 plus damages.
As builders in bad faith, respondents have no right to recover their expenses over the
improvements they have introduced to petitioners' lot under Article 449 of the Civil Code, which
provides:
Article 449. He who builds, plants or sows in bad faith on the land of another, loses what is built,
planted or sown without right to indemnity.
Therefore, petitioners as landowners became the owners of the improvements on the lot, including
the residential buildings constructed by respondents, if they chose to appropriate the accessions.
However, they could instead choose the demolition of the improvements at respondents' expense
or compel respondents to pay the price of the land under Article 450 of the Civil Code, which
provides:
Article 450. The owner of the land on which anything has been built, planted or sown in bad faith
may demand the demolition of the work, or that the planting or sowing be removed, in order to
replace things in their former condition at the expense of the person who built, planted or sowed; or
he may compel the builder or planter to pay the price of the land, and the sower the proper rent.
PROPERTY Digested by Jennylyn C. Mondejar

34. Pen Development vs Leyba Inc


GR No. 211845 August 9, 2017
Facts:
Leyba Inc. is a corporation owned and operated by Martinez, which owns 3 contiguous parcels of
land in Antipolo Rizal identified as Lots 29, 30 and 31. Martinez noticed that the construction of Las
Brisas' fence seemed to encroach on its land. Upon verification by surveyors, Martinez was
informed that the fence of Las Brisas overlaps its property. Martinez sent a Letter informing Las
Brisas that the fence it constructed encroaches on Martinez's land and requested Las Brisas to
refrain from further intruding on the same.
Pen Development Corporation and Las Brisas Resorts Corporation also domestic corporations,
merged into one corporate entity. Las Brisas did not respond to Martinez's letter demanding the
latter to cease and desist from unlawfully holding portions of his land and continued developing its
land.
Martinez filed a Complaint for Quieting of Title, Cancellation of Title and Recovery of Ownership
with Damages.
Issue:
Whether Las Brisas the defendant is a builder in bad faith and is liable for the consequence of their
acts.
Ruling:
YES. Las Brisas is a builder in bad faith. Under the Civil Code, Art. 449. He who builds, plants or
sows in bad faith on the land of another, loses what is built, planted or sown without right to
indemnity.
Art. 450. Tue owner of the land on which anything has been built, planted or sown in bad faith may
demand the demolition of the work, or that the planting or sowing be removed, in order to replace
things in their former condition at the expense of the person who built, planted or sowed; or he may
compel the builder or planter to pay the price of the land, and the sower the proper rent.
Art. 451. In the cases of the two preceding articles, the landowner is entitled to damages from the
builder planter or sower.
Moreover, it has been declared that the right of the owner of the land to recover damages from a
builder in bad faith is clearly provided for in Article 451 of the Civil Code.
In this case, petitioners are not entitled to reimbursement for necessary expenses.
PROPERTY Digested by Jennylyn C. Mondejar

Articles 452-454

35. G.R. No. L-9438 PAULA MARTINEZ vs VICTORINO BAGANUS

Facts: Paula Martinez was an owner of a town lot. His son, Jose Mojica sold the lot to Victorino
Baganus for P70. Subsequently, Martinez filed a case seeking recovery of ownership and
possession of the lot alleging that the sale made by his son Mojica is null and void.

The defendant Baganus alleged that he sough the lot from Jose Mojica with the consent of the
latter's mother, Paula Martinez, in the belief that it belonged to Jose Mojica and that the money
with which the latter had bought it had been earned by Jose when he was the patron of a sailboat;
that he had paid to Jose Martinez a deposit of P30 in advance and agreed to pay the P40 balance
when the instrument of sale had been executed, as was later done; that, confiding in good faith in
the validity and efficacy of his acquisition, he had made necessary and useful improvements on the
lot, having a house and a carriage shop thereon, graded it, and planted fruit trees, all which had
costs him over P1,000.

Issue: Whether or not the Baganus acquired the land in bad faith.

Ruling:

Yes. Baganus acquired the lot in bad faith, for he himself said that he dealt with Martinez but later
on consented that Mojica, who was not the owner should appear as the vendor. However, Martinez
was also in bad faith because the act of building and planting has been executed in her presence
with her knowledge and tolerance and without objection.

Article 453: If there is bad faith, not only on the part of the person who built, planted or sown on the
land of another, but also on the part of the owner of such land, the rights of one and the other shall
be the same as though both had acted in good faith.

It is understood that there is bad faith on the part of the landowner whenever the act was done with
his knowledge and without opposition on his part.

Being that there was bad faith on the part of Martinez, the owner and Baganus, the builder and
planter. The bad faith of one cancels the bad faith of the other.

Hence, in this case the court ordered: that Martinez should indemnify Baganus to the value of the
buildings and plantings he has placed thereon, with the right on his part to retain it until she has
reimbursed him for said necessary and useful improvements; or for Baganus to pay Martinez the
price of the land.
PROPERTY Digested by Jennylyn C. Mondejar

36. G.R. No. 120303 FEDERICO GEMINIANO vs CA

Facts: A 314 sq.m. lot was originally owned by the petitioners’ mother, Paulina Geminiano. On a
portion of the said lot stood the petitioners’ unfinished bungalow which they sold in 1978 to Sps.
Nicolas with an alleged promise to sell the lot occupied by the house. Subsequently, the petitioners’
mother executed a contract of lease over a 126 square meter portion of the lot, including that
portion on which the house stood, in favor of Sps. Nicolas (private respondents) for a period of
seven years.

The Sps. Nicolas then introduced additional improvements and registered the house in their
names. After the expiration of the lease in 1985, the petitioners’ mother refused to accept the
monthly rentals. It turned out that the lot in question was the subject of a suit, which resulted in its
acquisition by one Maria Lee in 1972. In 1982, Lee sold the lot to Lily Salcedo, who in turn sold it in
1984 to the spouses Agustin and Ester Dionisio. In 1992, the Dionisio spouses executed a Deed of
Quitclaim over the said property in favor of the petitioners. As such, the lot was registered in the
latter’s names. In 1993, the petitioners filed an ejectment case against Mary Nicolas.

Issue:
WON the lessees were builders in good faith and entitled to reimbursement of the value of the
houses and improvements

Ruling:

No. Being mere lessees, the private respondents knew that their occupation of the premises would
continue only for the life of the lease. Plainly, they cannot be considered as possessors nor
builders in good faith.

Anent the alleged promise of the petitioners to sell the lot occupied by the private respondents’
house, the same was not substantiated by convincing evidence. Neither the deed of sale over the
house nor the contract of lease contained an option in favor of the respondent spouses to
purchase the said lot. And even if the petitioners indeed promised to sell, it would not make the
private respondents possessors or builders in good faith so as to be covered by the provisions of
Article 448 of the Civil Code. The latter cannot raise the mere expectancy of ownership of the
aforementioned lot because the alleged promise to sell was not fulfilled nor its existence even
proven.

It must be stressed, however, that the right to indemnity under Article 1678 of the Civil Code arises
only if the lessor opts to appropriate the improvements. Since the petitioners refused to exercise
that option, the private respondents cannot compel them to reimburse the one-half value of the
house and improvements. Neither can they retain the premises until reimbursement is made. The
private respondents’ sole right then is to remove the improvements without causing any more
impairment upon the property leased than is necessary.
PROPERTY Digested by Jennylyn C. Mondejar

37. G.R. No. 117642 EDITHA ALVIOLA vs CA

Facts:
Victoria Tinagan and her son Agustin own 2 parcels of land. Sometimes in 1960, Victoria allowed
Spouses Alviola to build a coper dryer and put up a store on portions of the land. Both Victoria and
Agustin died; they were succeeded by the heirs of Agustin. The Heirs argued that, they are the
owners of the disputed properties; and that the occupation of the Spouses Alviola were only
allowed by mere tolerance. On the other hand, Spouses Alviola contended that the subject portions
of land were conveyed to them due to alleged debts owe by Augustin.

Issue:
WON petitioners were in bad faith in possessing the disputed properties and in ruling that the
improvements thereon are transferable.

Ruling:
Yes. There was bad faith on the part of the petitioners when they constructed the copra dryer and
store on the disputed portions since they were fully aware that the parcels of land belonged to
Victoria Tinagan. And, there was likewise bad faith on the part of the private respondents, having
knowledge of the arrangement between petitioners and Victoria Tinagan relative to the construction
of the copra dryer and store. Thus, for purposes of indemnity, Article 448 of the New Civil Code
should be applied. However, the copra dryer and the store, as determined by the trial court and
respondent court, are transferable in nature. Thus, it would not fall within the coverage of Article
448. As the noted civil law authority, Senator Arturo Tolentino, aptly explains: "To fall within the
provision of this Article, the construction must be of permanent character, attached to the soil with
an idea of perpetuity; but if it is of a transitory character or is transferable, there is no accession,
and the builder must remove the construction. The proper remedy of the landowner is an action to
eject the builder from the land.
PROPERTY Digested by Jennylyn C. Mondejar

38. DELOS SANTOS VS. ABEJON G.R. NO. 215820 | March 20, 2017

FACTS:
Erlinda and her late husband Pedro Delos Santos (Pedro) borrowed the amount of ₱100,000.00
from the former's sister, Teresita, as evidenced by a Promissory Note dated April 8, 1998 and as
security for the loan, Erlinda and Pedro mortgaged their property which mortgage was annotated
on the title.

After Pedro died, Erlinda ended up being unable to pay the loan, and as such, agreed to sell the
subject land to Teresita for ₱150,000.00 or for the amount of the loan plus an additional
₱50,000.00. In 1992, they executed a Deed of Sale and a Release of Mortgage, and eventually,
TCT No. 131753 was cancelled and TCT No. 180286 was issued in the name of "Teresita, Abejon,
married to Alberto S. Abejon.

The Abejons then constructed a three-storey building worth P2,000,000. The Delos Santos family
denied the 1992 sale since Pedro died in 1989. The Abejons sought P2,000,000 from the Delos
Santos family for the cost of the building, and P150,000 for the land before they would turn them
over. Delos Santos used the defense of not having participated in the spurious sale and claimed
that Teresita constructed the building voluntarily. RTC declared the sale void. Delos Santos family
ordered to pay for the loan, the building and the litigation costs and CA affirmed the RTC Ruling
with modifications.

ISSUE:

Whether or not both the landowner and builders be treated as if both of them were in good faith.

RULING:
Yes. Despite such awareness of the defect in their title to the subject land, respondents still
proceeded in constructing a three (3)-storey building thereon. Indubitably, they should be deemed
as builders in bad faith. On the other hand, petitioners knew of the defect in the execution of the
Deed of Sale from the start, but nonetheless, still acquiesced to the construction of the three (3)-
storey building thereon. Hence, they should likewise be considered as landowners in bad faith.In
this relation, Article 453 of the Civil Code provides that where both the landowner and the builder,
planter, or sower acted in bad faith, they shall be treated as if both of them were in good faith, viz.:
Article 453. If there was bad faith, not only on the part of the person who built, planted or sowed on
the land of another, but also on the part of the owner of such land, the rights of one and the other
shall be the same as though both had acted in good faith.
PROPERTY Digested by Jennylyn C. Mondejar

Article 457

1. GRANDE VS. CA G.R. NO. L-17652 | June 30, 1962

FACTS:

Petitioners are the owners of a parcel of land by inheritance from their deceased mother Patricia
Angui. When it was surveyed for purposes of registration, its northeastern boundary was the
Cagayan River (the same boundary stated in the title).
Since then, and for many years thereafter, a gradual accretion on the northeastern side took place,
by action of the current of the Cagayan River.
Petitioners instituted the present action in the Court of First Instance of Isabela against
respondents, to quiet title to said portion (19,964 square meters) formed by accretion, alleging in
their complaint that they and their predecessors-in-interest, were formerly in peaceful and
continuous possession thereof, until September, 1948, when respondents entered upon the land
under claim of ownership. In their answer, respondents claim ownership in themselves, asserting
that they have been in continuous, open, and undisturbed possession of said portion, since prior to
the year 1933 to the present.

ISSUE:
WON respondents have acquired the alluvial property in question through prescription.

RULING:
Yes. The Supreme Court aid that ownership is one thing under the principle of accession. You are
automatically the owner of anything produced or attached to your property. But if the Civil Code
says that you have to register but you did not so you will lose the ownership.
The fact remains, however, that the Grandes never sought registration of said alluvial property
(which was formed sometime after Grande's property was registered on June 9, 1934) up to the
time they instituted the present action in 1958. The increment, therefore, never became registered
property, and hence is not entitled or subject to the protection of imprescriptibility enjoyed by
registered property under the Torrens system. Consequently, it was subject to acquisition through
prescription by third persons.
In other words, if this registered property was the one occupied by X, X will never acquire
ownership and C will never lose ownership. Unfortunately for C, what was occupied by X was the
portion formed by accretion which was not registered in time.
PROPERTY Digested by Jennylyn C. Mondejar

2. ZAPATA VS. DIRECTOR OF LANDS G.R. NO. L-17645 | October 30, 1962

FACTS:
Juliana Zapata owns two parcels of land situated in the municipality of Santo Tomas, province of
Pampanga, adjoining a non-navigable and non-floatable river called the Candalaga Creek and
these two parcels are designated as Lot No. 25 and the northern part of Lot No. 16 of the
Cadastral Survey of San Fernando, Pampanga.
The first lot is registered in her name and her ownership or title to a part of Lot No. 16 was
confirmed by a decree entered on 21 November 1955 by the Court of First Instance of Pampanga
ordering that the "remaining portion of Lot No. 16 with an area of 474 square meters" be registered
"in the name of Juliana Zapata". At present, the width of the Candalaga Creek adjoining the two
parcels of land owned by Juliana Zapata is 15 meters because soil had been accumulated by the
water current of the river on the banks of Lot No. 25 and of that part of Lot No. 16 owned by
Juliana Zapata. In a verified petition, Juliana Zapata claims that the aforesaid three lots belong to
her by accretion, was provided for in article 457 of the Civil Code, and prays that the same be
registered in her name under the Land Registration Act.
On 19 October 1956 on her motion the court entered an order of general default against all
persons except the Director of Lands. On 24 October 1956 the Director of Lands objected to the
petition and prayed that the registration of the three lots in the name of Jualiana Zapata be denied
and that they be declared to form part of the public domain. he appellant claims that the accretion
was artificially brought about by the setting up of fish traps, such as salag net, bunuan (Bamboo
trap), sabat (cutting of channels) and fencing that the fishermen had built in the stream which was
not supported by Article 457.

ISSUE:
WON the aforesaid three lots belong to Juliana Zapata by accretion.

RULING:
Yes. Despite the fact that the fish traps actually helped perhaps in the trapping of the soil and not
only the fish. So the river pushed the soil in the land and the fish traps prevented from coming back
to the river. The SC said that maybe the fish traps helped but it was not only the reason why
alluvial deposits were made on the property of Zapata.
As there is no evidence to show that the setting up or erection of the fish trap was expressly
intended or designed to cause or bring about the accretion, Zapata may still invoke the benefit of
the provisions of Article 457 of the Civil Code.
In this 1962 case, SC said that as long as the riparian owner did not set up any gadget in order to
attract the soil so that his land will become bigger, then he can still benefit under Article 457.
PROPERTY Digested by Jennylyn C. Mondejar

3. Agustin v. IAC GR NO. 66075-76 Jul 5, 1990

Facts:

In 1919 the lands east of the river were covered by the Tuguegarao Cadastre. As the years went
by, the Cagayan River moved gradually eastward, depositing silt on the western bank. The shifting
of the river and the siltation continued until 1968. However, after a big flood, the Cagayan River
changed its course, returned to its 1919 bed, and, in the process, cut across the lands of Melad,
Binayug and Ubina whose lands were transferred on the eastern, or Tuguegarao, side of the river.
To cultivate those lots they had to cross the river. In April, 1969, while Melad, Binayug and Ubina
were planting corn on their lots located on the eastern side of the Cagayan River, Agustin
accompanied by the mayor and some policemen of Tuguegarao, claimed the same lands as their
own and drove away the Melad, Binayug and Ubina from the premises. This prompted Melad,
Binayug and Ubina to file a case to recover their lots and accretions.

Issue:
WON Agustin is the rightful owner of the said property.

Held:

No, Gradually, the river moved for almost 50 years. Melad in this case still owns the accretion by
virtue of Article 457. Articles 461 was applied in this case. Melad owns the dried up river bed.

The rightful owners are Binayug and Melad since they became the owners of the alluvium from the
effects of the gradual movement of the river from. The court held that if the lots of the riparian
owner are exposed to flood and other damage due to the destructive force of water, it would be just
that risk and damaged be compensated with the Right of Accretion. The private respondents'
ownership of the accretion to their lands was not lost upon the sudden and abrupt change of the
course of the Cagayan River in 1968 or 1969 when it reverted to its old 1919 bed, and separated
or transferred said accretions to the other side (or eastern bank) of the river. Articles 459 and 463
of the New Civil Code apply to this situation.In the case at bar, the sudden change of course of the
Cagayan River as a result of a strong typhoon in 1968 caused a portion of the lands of the private
respondents to be "separated from the estate by the current." The private respondents have
retained the ownership of the portion that was transferred by avulsion to the other side of the river.
PROPERTY Digested by Jennylyn C. Mondejar

4. Vda. De Nazareno v. CA GR NO. 98045 Jun 26, 1996

Facts:
The subject of this controversy is a parcel of land to which it was formed as a result of the dumping
of sawdust, boulders, soil, and other filling materials into the dried-up Balacanas creek along the
banks of the Cagayan River by the Sun Valley Lumber Co. as a consequence to their sawmill
operations.
The controversy started when an opposition was made by Salasan and Rabaya when Antonio
Nazareno wanted to perfect his title over the accretion area. According to the Bureau of Lands,
however, the subject land was a public land being a man-made accretion.

This was questioned by the Nazarenos arguing that the subject land is private land being an
accretion to his titled property pursuant to Article 457 of the Civil Code.
e said plan claiming that they should be the ones to whom the accretion should be awarded.

Issue:
WON the accretion which was formed by sawdust being dumped can be claimed by the riparian
owner pursuant to Art 457 of the Civil Code

Held:

No, the court held that accretion, as a mode of acquiring property under Art. 457 of the Civil Code,
must contain the following requisites:
(1) that the deposition of soil or sediment be gradual and imperceptible;
(2) that it be the result of the action of the waters of the river (or sea); and
(3) that the land where accretion takes place is adjacent to the banks or rivers (or the sea coast).

In this case, the accretion was formed by the dumping of boulders, soil, and other filling materials on portions of
the Balacanas Creek and the Cagayan River bounding their land. Therefore, such accumulation of boulders, soil,
and other filling materials was not gradual and imperceptible, and not resulting from the action of the waters of
the current of the Balacanas creek. It follows, then, that the accretion was man-made or artificial.

Not having met the first and second requisites of the rules on alluvion, they cannot claim the rights of a riparian
owner. The Supreme Court, furthermore, emphasized in this case that in order for Article 457 of the Civil Code
to apply, alluvion must be the exclusive work of nature, that is without any direct or deliberate human
intervention. Therefore, if such accretion was man-made, it is deemed part of the public domain.
PROPERTY Digested by Jennylyn C. Mondejar

5. Heirs of Navarro v. IAC GR NO. 68166 Feb 12, 1997

Facts:

Sinforoso Pascual was an owner of a tract of land located at Barangay Puerto Rivas, Balanga, Bataan. This land
is bounded on the North by the Manila Bay, on the East by the Talisay river and on the West by the Bulacan
river. Pascual planted palapat and bakawan trees on the northern boundary of his property and years later, the
land begun to rise.

Pascual filed an application to register and confirm his title to the accretion which formed at the northern portion
of his property. Pascual contends that the accretion was formed by the actions of the Bulacan and Talisay River
and as the riparian owner, he clams ownership thereof.

Navarro opposed Pascual’s application contending that the land sought to be registered is part of the public
domain, it being part of Manila Bay

Issue: Whether or not Pascual can acquire the subject land by Accretion under Art. 457.

Held:

No. The disputed property was brought forth by both the withdrawal of the waters of Manila Bay
and the accretion formed on the exposed foreshore land by the action of the sea which brought soil
and sand sediments in turn trapped by the palapat and bakawan trees planted thereon by Pascual
in 1948. Here, Pascual anchored his claim of ownership on Article 457 of the Civil Code.

He argued that the disputed land is an accretion caused by the joint action of the Talisay and
Bulacan River which run their course on the eastern and western boundaries respectively of
Pascual’s own tract of land. In this case, the 3rd requisite under Art 457 is lacking, which is, that
the alluvium is deposited on the portion of claimant’s land which is adjacent to the river bank.

In this case, the disputed land, is an accretion not on a river bank but by a sea bank, or on what
used to be the foreshore of Manila Bay which adjoined Pascual’s own tract on the northern side. As
such, the applicable law is not Art. 457 of the NCC but Art. 4 of the Spanish Law of Waters of 1866.
Therefore, Pascual cannot acquire and register the subject property in his name because the
increased land adjoining to his registered land was adjoined and formed by the action of the Manila
Bay (Sea) and not by the 2 rivers in this case, thus, Art. 457 not applicable.
PROPERTY Digested by Jennylyn C. Mondejar

6. Bagaipo v. CA GR NO. 116290 Dec 8, 2000

FACTS:
Dionisia P. Bagaipo is the registered owner of Lot No. 415, agricultural land situated in Ma-a,
Davao City. Leonor Lozano is the owner of a registered parcel of land located across and opposite
the southeast portion of Bagaipo’s lot facing the Davao River. Lozano acquired and occupied his
property in 1962 when his wife inherited the land from her father who died that year.

On May 26, 1989, Bagaipo filed a complaint for Recovery of Possession with Mandatory Writ of
Preliminary Injunction and Damages against Lozano for the surrender of possession by Lozano of
a certain portion of land measuring 29,162 square meters which is supposedly included in the area
belonging to Bagaipo; recovery of a land area measuring 37,901 square meters which Bagaipo
allegedly lost when the Davao River traversed her property.

For his part, Lozano insisted that the land claimed by Bagaipo is actually an accretion to their titled
property.The Davao River did not change its course and that the reduction in Bagaipo’s domain
was caused by gradual erosion due to the current of the Davao River. It is also because of the
river’s natural action that silt slowly deposited and added to his land over a long period of time.

ISSUE:

WON Lozano owns Lot 415-C in accordance with the principle of accretion under Article 457.

RULING:

YES. Lozano owns Lot 415-C in accordance with the principle of accretion. When accretion
1
benefits a riparian owner. The rule is well-settled that accretion benefits a riparian owner when the
following requisites are present:

1. that the deposit be gradual and imperceptible;

2. that it resulted from the effects of the current of the


water; and

3. that the land where accretion takes place is adjacent to


the bank of the river.

These requisites were sufficiently proven in favor of Lozanos. In the absence of evidence that the
change in the course of the river was sudden or that it occurred through avulsion, the presumption
is that the change was gradual and was caused by alluvium and erosion. The fact that the
accretion to his land used to pertain to plaintiff’s estate, which is covered by a Torrens certificate of
title, cannot preclude him (defendant) from being the owner thereof. Registration does not protect
the riparian owner against the diminution of the area of his land through gradual changes in the
course of the adjoining stream. Bagaipo did not demonstrate that Lot 415-C allegedly comprising
29,162 square meters was within the boundaries of her titled property. The survey plan was also
not approved by the Director of Lands, hence discounted by the Court.
PROPERTY Digested by Jennylyn C. Mondejar

7. Siain Enterprises v. FF Cruz GR NO.146616 Aug 31, 2006

FACTS:

In 1973, WESVICO filed a foreshore lease application over the foreshore land adjacent to certain
lots registered in its name. It eventually withdrew such application and filed instead a petition for
registration over the same foreshore land. The case however did not again push through.

Later, WESVICO’s properties including Lot 3309 were foreclosed by DBP. SIAIN Ent. purchased it.

In 1983, FF Cruz filed a foreshore lease application over a foreshore land, a portion of which is
adjacent to Lot 3309. In the investigation, it was found that FF Cruz was already occupying and
using the area and that they even filled up a portion of the land.

In 1986, SIAIN also filed a foreshore lease application over the foreshore land adjacent to the
properties it bought from DBP. It was found that 130 linear meters of the foreshore land subject of
FF Cruz’s application overlapped that applied for by SIAIN. So, SIAIN filed a protest alleging that it
should be given preference in its lease because it was the owner of the property adjoining the
overlapping area. FF Cruz on the other hand alleged that it submitted the application first.

The Land Management Bureau decided that the subject area was not a natural foreshore but
instead it is a land reclaimed by FF Cruz.

ISSUE:

WON SIAIN has preferential right to lease the same.

RULING:

YES. That the foreshore area had been reclaimed does not remove it from its classification of
foreshore area subject to the preferential right to lease of the littoral owner.

Take note that it was not the reclamation that brought the disputed foreshore area into existence.
Such foreshore area existed even before F.F. Cruz undertook its reclamation. It was "formed by
accretions or alluvial deposits due to the action of the sea" and SIAIN, being the littoral owner has
preferential right to lease the same.

Contrary to the allegation that WESVICO, SIAIN’s predecessor had already waived its preferential
right, the Court ruled that it cannot be considered to have waived or abandoned its preferential
right to lease the disputed area when it subsequently filed an application for registration thereover.

Being a foreshore land, it was therefore a part of the public domain and ownership of the area
could not be acquired by WESVICO. It does not mean however that it lost or waived its preferential
right. The right remains and may now be invoked by SIAIN being the new owner of the property.
PROPERTY Digested by Jennylyn C. Mondejar

8. Office of City Mayor v. Ebio GR NO. 178411 Jun. 23, 2010

FACTS:

There is this land occupied by Spouses Ebio which is an alluvium of Cut- cut Creek.This alluvium is
located beside Road Lot No. 8 which is owned by Guaranteed Homes, Inc. The road lot was later
donated to the City government of Parañaque on 1966.

On 1999, the City Government of Parañaque decided to construct an access road along the Cut-
cut Creek which would traverse thealluviumproperty. Mario Ebio, the husband, opposed such
construction by asserting their claim of ownership of the said property. Heasserted that he and his
predecessors-in-interest has been in the possession of the property since 1930 and they had
introduced improvements thereon. The City Goverment, however, argued that since the creek is
classified as part of the public domain, any land that may have formed along its banks through time
should also be considered as part of the public domain. So, the City government sent a letter to the
spouses Ebio ordering them to vacate the area or be physically evicted. This prompted the
spouses to go to the Regional Trial Court to stop the construction of the access road.Unfortunately,
the Regional Trial Court denied their prayer saying that they were not able to prove that they have
an established right to the property since their application for sales patent has not yet been
granted.

ISSUE:

WON the spouses Ebio have right of ownership over the property

RULING: Yes.

The spouses Ebio have right of ownership over the property.

Being alluvial deposits that have gradually settled along the banks of Cut-cut creek, the law that
governs ownership over this accreted land are Article 84 of the Spanish Law of Waters of 1866 and
Article 457 of the Civil Code.

To the owners of lands adjoining the banks of rivers belong the accretion which they gradually
receive from the effects of the current of the waters.

It is therefore explicit from these provisions that these deposits do not form part of the public
domain but are private properties which automatically belong to the owner of the estate to which it
may have been added.

In the case at bar, Mario Ebio established that he and his predecessors- in-interest has
beenoccupying and possessing the subject lot as early as 1930, as evidenced by their tax
declarations. Also, for more than 30 years, neither Guaranteed Homes, Inc. nor the local
government of Paranaque sought to register the accreted portion. By this failure, the accreted
portion was subject to acquisitive prescription in which spouses Ebio based their right.

Therefore, Spouses Ebio is deemed to have acquired ownership over the subject property.
Spouses Ebio can assert such right despite the fact that they had yet to register their title over the
said lot. Registration was never intended as a means of acquiring ownership. Its purpose is the
registration of title which the applicant already possesses.
PROPERTY Digested by Jennylyn C. Mondejar

9. DELOS REYES AND PERALTA V. MUNICIPALITY OF KALIBO GR No. 214587, February 26,
2018

FACTS

Although Ana Peralta was the registered owner of Lot No. 2076 of the Kalibo Cadastre, the same
land was also registered to her brother Jose Peralta who divided the lot into Lot No. 2076-A, which
remained to him, and Lot No. 2076-B, which he sold.

Regarding Lot No. 2076, it had allegedly acquired accretion, and this accretion was occupied and
declared in a tax declaration by Ambrocio Ignacio, a tenant of the Peraltas, who eventually
executed a Quitclaim of Real Property in favor of Jose.

Upon Jose’s death, Lot No. 2076-A was transferred to his son Juanito Peralta, but the accretion
was registered in tax declarations under his name along with his siblings Javier Peralta, Josephine
delos Reyes, and Julius Peralta (Petitioners).

The Municipality of Kalibo (Respondent) wanted to covert a portion of the accretion area into a
garbage dumpsite, and despite the Peralta siblings’ objection, the Municipality pushed through the
project contending that the accretion was of public dominion.

The RTC, upon the Peraltas’ Complaint for quieting of title over the accretion, decided in favor of
them, declaring that the 2 portions of accretion were not public land, thus the Municipality cannot
covert such into a dumpsite.

The CA reversed the RTC ruling and decided in favor of the Municipality.

ISSUE

WON the 2 portions of land were accretions which can be considered owned by the Peraltas

RULING

NO, Article 457 of the New Civil Code provides that “to the owners of lands adjoining the banks of
rivers belong the accretion which they gradually receive from the effects of the current waters.”

Accretion is the process whereby the soil is deposited along the banks of rivers, by which the
deposit of soil considered as accretion must be: (a) gradual and imperceptible; (b) made through
the effects of the current of the water; and (c) taking place on land adjacent to the banks of rivers.

In this case, it was found that the increments on the subject area were predominantly composed of
sand deposits rather than soil deposits.

Thus, the increments on the land are considered part of public domain which allows the
Municipality to covert it to a garbage dumpsite and bars the transfer by Jose Peralta to his children
(herein Petitioners).
PROPERTY Digested by Jennylyn C. Mondejar

ARTICLE 458

10. GOVERNMENT V. COLEGIO DE SAN JOSE 53 Phil 423, GR No. L-30829, August 29, 1929

FACTS

Involved were 2 parcels of land Lots 1 and 2 whose eastern borders were beside a flooded strip of
land covered by Laguna de Bay during September, October, and November, the land strip
including a pass claimed by the Municipality of San Pedro Tunasan.

Colegio de San Jose (Claimant) contended that the lots were part of Hacienda de San Pedro
Tunasan and that these were in their possession ever since by way of their tenants and farmers.

The Government of the Philippine Islands argued that the lots were of public domain as it has
always been the “shores of Laguna de Bay”.

ISSUE:

WON the 2 parcels of land belong to Colegio de San Jose

RULING

YES, the Court held that the said parcels are outside the bed of Laguna de Bay, thus not belonging
to the public domain.

Article 458 of the New Civil Code (Article 367 in the Civil Code) cannot be applied in this case as
contended by the Government, which provides that “the owners of estates adjoining ponds or
lagoons do not acquire the land left dry by the natural decrease of the waters, or lose that
inundated by them in extraordinary floods.”

In this case, Laguna de Bay was found to be a lake, not a lagoon which is referred to in Article 458,
since a lagoon is legally distinct in character from a lake. The lots in question were also only
“accidentally inundated by the waters of Laguna de Bay”, that is being accidentally flooded by the
subject lake.

Consequently, the 2 parcels of land in litigation must belong to Colegio de San Jose as a part of
Hacienda de San Pedro Tunasan, and the fact that the lands were flooded by the lake’s waters
during extraordinary risings does not deprive claimant of ownership.
PROPERTY Digested by Jennylyn C. Mondejar

ARTICLE 461

11. SPS. BAES V. CA AND REPUBLIC GR No. 108065, July 6, 1993

FACTS

Lot 2958 was dug by the government, making a man-made canal to restructure the Tripa de
Gallina creek and was later on acquired by Felix Baes who subdivided the lot into 3 parts: Lot
2958-A, Lot 2958-B, and Lot 2958-C.

As Lot 2958-B was occupied by the canal, the government gave Baes Lot 3271-A by virtue of a
Deed of Exchange of Real Property – a lot that has the exact same area size as that of the former
lot situated near Lot 2958-C.

Baes eventually had Lot 2958-C and a portion of Lot 2958-A resurveyed and subdivided as there
were errors in the plans which resulted to new titles on Lot 1-A, Lot 1-B (increase in area after
resurvey), Lot 2958-C-1, and Lot 2958-C-2 (increase in area after resurvey), the latter two being
further subdivided into four.

The Republic of the Philippines (Respondent) later discovered that Lot 1-B is a filled portion of the
creek and was where Spouses Felix and Rafaela Baes (Petitioners) built an apartment and that Lot
2958-C was unlawfully enlarged.

On trial, the trial court ordered that Lot 2958-C shall be reverted back to its status before the
resurvey, but Lot 1-B, an increment, was still on dispute to which Sps. Baes averred that the said
lot was theirs in accordance with Article 461 of the New Civil Code.

ISSUE:
WON Article 461 shall apply to this case

RULING

YES, Article 461 of the New Civil Code provides that “river beds which are abandoned through the
natural change in the course of the waters ipso facto belong to the owners whose lands are
occupied by the new course in proportion to the area are lost. However, the owners of the lands
adjoining the old bed shall have the right to acquire the same by paying the value thereof, which
shall not exceed the value of the area occupied by the new bed.”

Moreover, If the riparian owner is entitled to compensation for the damage to or loss of his property
due to natural causes, there is all the more reason to compensate him when the change in the
course of the river is effected through artificial means. According to Dr. Arturo Tolentino, if the
change of the course is due to works constructed by concessioners authorized by the government,
the concession may grant the abandoned riverbed to concessioners, but if there is no such grant,
then the abandoned riverbed will belong to the owners of the land covered by the waters without
prejudice to a superior right of third persons with sufficient title.

In this case, the loss of Sps. Baes of the land covered by the canal was the result of a deliberate
act on the part of the government when it wanted to improve the flow of Tipa de Gallina creek, thus
obligating the government to compensate the spouses for the loss to which the government
already did by giving the spouses Lot 3271-A as an exchange for Lot 2958-B.

You might also like